|
|
| (26 intermediate revisions by 15 users not shown) |
| Line 1: |
Line 1: |
| Agnes writes the following four statements on a blank piece of paper.
| | #redirect [[2025 AMC 12A Problems/Problem 4]] |
| | |
| <imath>\bullet</imath> At least one of these statements is true.
| |
| | |
| <imath>\bullet</imath> At least two of these statements are true.
| |
| | |
| <imath>\bullet</imath> At least two of these statements are false.
| |
| | |
| <imath>\bullet</imath> At least one of these statements is false.
| |
| | |
| Each statement is either true or false. How many false statements did Agnes write on the paper?
| |
| | |
| <imath>\textbf{(A) } 0 \qquad\textbf{(B) } 1 \qquad\textbf{(C) } 2 \qquad\textbf{(D) } 3 \qquad\textbf{(E) } 4</imath>
| |
| | |
| ==Solution 1==
| |
| | |
| We first number all the statements:
| |
| | |
| 1) At least one of these statements is true.
| |
| 2) At least two of these statements are true.
| |
| 3) At least two of these statements are false.
| |
| 4) At least one of these statements is false. | |
| | |
| We can immediately see that statement 4 must be true, as it would contradict itself if it were false. Similarly, statement 1 must be true, as all the other statements must be false, which is established to be false by statement 4. As both 1 and 4 are True, statement 2 is true. Therefore, statement 3 is the only false statement, as there are three true statements, making the answer <imath>\boxed{\text{(B) }1}</imath>.
| |
| -Rainjs
| |
| | |
| ==Solution 2==
| |
| Statements <imath>I,II,</imath> and <imath>IV</imath> are true, while statement <imath>III</imath> is false. Hence, there are <imath>3</imath> true statements and <imath>\boxed{\text{(B) }1}</imath> false statement. This result can be checked by examining the statements individually again.
| |
| | |
| Statements <imath>I</imath> and <imath>II</imath> will be true because there are <imath>3\ge2</imath> true statements. Statement <imath>IV</imath> is also true because there is <imath>1\ge1</imath> false statement. Finally, statement <imath>III</imath> is false because there are <imath>1\ngeq2</imath> false statements.
| |
| | |
| ~Tacos_are_yummy_1
| |